Difference between revisions of "2018 AMC 10B Problems/Problem 23"

(Original had 21*8 = 189 which is not true. Also, fixed the latex formatting.)
(Video Solution)
 
(27 intermediate revisions by 18 users not shown)
Line 1: Line 1:
 +
==Problem==
 +
 
How many ordered pairs <math>(a, b)</math> of positive integers satisfy the equation  
 
How many ordered pairs <math>(a, b)</math> of positive integers satisfy the equation  
 
<cmath>a\cdot b + 63 = 20\cdot \text{lcm}(a, b) + 12\cdot\text{gcd}(a,b),</cmath>
 
<cmath>a\cdot b + 63 = 20\cdot \text{lcm}(a, b) + 12\cdot\text{gcd}(a,b),</cmath>
Line 7: Line 9:
  
 
==Solution==
 
==Solution==
Let <math>x = \lcm(a, b)</math>, and <math>y = \gcd(a, b)</math>. Therefore, <math>a\cdot b = \lcm(a, b)\cdot gcd(a, b) = x\cdot y</math>. Thus, the equation becomes
+
Let <math>x = \text{lcm}(a, b)</math>, and <math>y = \text{gcd}(a, b)</math>. Therefore, <math>a\cdot b = lcm(a, b)\cdot gcd(a, b) = x\cdot y</math>. Thus, the equation becomes
  
 
<cmath>x\cdot y + 63 = 20x + 12y</cmath>
 
<cmath>x\cdot y + 63 = 20x + 12y</cmath>
 
<cmath>x\cdot y - 20x - 12y + 63 = 0</cmath>
 
<cmath>x\cdot y - 20x - 12y + 63 = 0</cmath>
  
Using Simon's Favorite Factoring Trick, we rewrite this equation as
+
Using [[Simon's Favorite Factoring Trick]], we rewrite this equation as
  
 
<cmath>(x - 12)(y - 20) - 240 + 63 = 0</cmath>
 
<cmath>(x - 12)(y - 20) - 240 + 63 = 0</cmath>
 
<cmath>(x - 12)(y - 20) = 177</cmath>
 
<cmath>(x - 12)(y - 20) = 177</cmath>
  
Since <math>177 = 3\cdot 59</math> and <math>x > y</math>, we have <math>x  - 12 = 59</math> and <math>y - 20 = 3</math>, or <math>x - 12 = 177</math> and <math>y - 20 = 1</math>. This gives us the solutions <math>(71, 23)</math> and <math>(189, 21)</math>. Obviously, the first pair does not work. Assume <math>a>b</math>. We must have <math>a = 21 \cdot 9</math> and <math>b = 21</math>, and we could then have <math>a<b</math>, so there are <math>\boxed{2}</math> solutions.
+
Since <math>177 = 3\cdot 59</math> and <math>x > y</math>, we have <math>x  - 12 = 59</math> and <math>y - 20 = 3</math>, or <math>x - 12 = 177</math> and <math>y - 20 = 1</math>. This gives us the solutions <math>(71, 23)</math> and <math>(189, 21)</math>. Since the <math>\text{GCD}</math> must be a divisor of the <math>\text{LCM}</math>, the first pair does not work. Assume <math>a>b</math>. We must have <math>a = 21 \cdot 9</math> and <math>b = 21</math>, and we could then have <math>a<b</math>, so there are <math>\boxed{\textbf{(B)} ~2}</math> solutions.
 
(awesomeag)
 
(awesomeag)
  
Edited by IronicNinja~
+
Edited by IronicNinja, Firebolt360, and mprincess0229~
 +
 
 +
== Video Solution by OmegaLearn==
 +
https://youtu.be/zfChnbMGLVQ?t=494
 +
 
 +
~ pi_is_3.14
 +
 
 +
==Video Solution==
 +
 
 +
https://www.youtube.com/watch?v=JWGHYUeOx-k
  
 
==See Also==
 
==See Also==
 
{{AMC10 box|year=2018|ab=B|num-b=22|num-a=24}}
 
{{AMC10 box|year=2018|ab=B|num-b=22|num-a=24}}
 
{{MAA Notice}}
 
{{MAA Notice}}

Latest revision as of 04:18, 21 January 2023

Problem

How many ordered pairs $(a, b)$ of positive integers satisfy the equation \[a\cdot b + 63 = 20\cdot \text{lcm}(a, b) + 12\cdot\text{gcd}(a,b),\] where $\text{gcd}(a,b)$ denotes the greatest common divisor of $a$ and $b$, and $\text{lcm}(a,b)$ denotes their least common multiple?

$\textbf{(A)} \text{ 0} \qquad \textbf{(B)} \text{ 2} \qquad \textbf{(C)} \text{ 4} \qquad \textbf{(D)} \text{ 6} \qquad \textbf{(E)} \text{ 8}$


Solution

Let $x = \text{lcm}(a, b)$, and $y = \text{gcd}(a, b)$. Therefore, $a\cdot b = lcm(a, b)\cdot gcd(a, b) = x\cdot y$. Thus, the equation becomes

\[x\cdot y + 63 = 20x + 12y\] \[x\cdot y - 20x - 12y + 63 = 0\]

Using Simon's Favorite Factoring Trick, we rewrite this equation as

\[(x - 12)(y - 20) - 240 + 63 = 0\] \[(x - 12)(y - 20) = 177\]

Since $177 = 3\cdot 59$ and $x > y$, we have $x  - 12 = 59$ and $y - 20 = 3$, or $x - 12 = 177$ and $y - 20 = 1$. This gives us the solutions $(71, 23)$ and $(189, 21)$. Since the $\text{GCD}$ must be a divisor of the $\text{LCM}$, the first pair does not work. Assume $a>b$. We must have $a = 21 \cdot 9$ and $b = 21$, and we could then have $a<b$, so there are $\boxed{\textbf{(B)} ~2}$ solutions. (awesomeag)

Edited by IronicNinja, Firebolt360, and mprincess0229~

Video Solution by OmegaLearn

https://youtu.be/zfChnbMGLVQ?t=494

~ pi_is_3.14

Video Solution

https://www.youtube.com/watch?v=JWGHYUeOx-k

See Also

2018 AMC 10B (ProblemsAnswer KeyResources)
Preceded by
Problem 22
Followed by
Problem 24
1 2 3 4 5 6 7 8 9 10 11 12 13 14 15 16 17 18 19 20 21 22 23 24 25
All AMC 10 Problems and Solutions

The problems on this page are copyrighted by the Mathematical Association of America's American Mathematics Competitions. AMC logo.png